Đến nội dung

Hình ảnh

bất đẳng thức


  • Please log in to reply
Chủ đề này có 8 trả lời

#1
Giang1994

Giang1994

    C'est la vie

  • Thành viên
  • 249 Bài viết
1.a,b,c dương. CmR:
$(a+ \dfrac{1}{b}-1)(b+\dfrac{1}{c}-1) +(b+\dfrac{1}{c}-1)(c+\dfrac{1}{a}-1)+ (c+\dfrac{1}{a}-1)(a+\dfrac{1}{b}-1)\geq3$
2.a,b,c,d dương CmR:
$ \sum \dfrac{a^{n+1}}{a^n+(n-1)b^n} \geq \dfrac{a+b+c+d}{n}$
$x_i$ dương chứng minh
3.$ \sum\limits_{i=1}^{n}x_{i}^{n+1}\geq x_1x_2...x_n(x_1+x_2+...+x_n)$
--------------
đừng chê dễ quá ha!
--------------
:(:):(:(:(

Bài viết đã được chỉnh sửa nội dung bởi Giang1994: 27-03-2011 - 13:45

Don't let people know what you think


#2
SLNA

SLNA

    Bảo Duyên

  • Thành viên
  • 77 Bài viết
Bài này của GS Vasile Cirtoaje :)

#3
h.vuong_pdl

h.vuong_pdl

    Thượng úy

  • Hiệp sỹ
  • 1031 Bài viết

3.$ \sum\limits_{i=1}^{n}x_{i}^{n+1}\geq x_1x_2...x_n(x_1+x_2+...+x_n)$
--------------
đừng chê dễ quá ha!
--------------
:(:):(:(:(

híc, toàn bài trâu bò thế này mà bảo là chê dễ, híc :((
Có lẽ bài 3 là dễ nhất, mình chém trước nha:
${\color{green} n^n(\sum\limits_{i=1}^{n}x_{i}^{n+1}) = (1+1+...+1)(1+1+...+1)...(1+1+...+1)(\sum\limits_{i=1}^{n}x_{i}^{n+1}) \\ \ge n^n.x_1.x_1.x_3....x_n(x_1+x_2+...+x_n) \to \textup{dpcm!}}$

rongden_167


#4
h.vuong_pdl

h.vuong_pdl

    Thượng úy

  • Hiệp sỹ
  • 1031 Bài viết
Chém luôn bài 2:
Sử dụng kĩ thuật Cô-si ngược dấu:
Ta có:
${\color{red}VT= \sum{\dfrac{a^{n+1}}(a^n+(n-1)b^n} = \sum{a-\dfrac{(n-1)a.b^n}{a^n+(n-1)b^n}} \ge \sum{a-\dfrac{(n-1).b}{n}} = \dfrac{a+b+c+d}{n}}$

p/s: bạn có thể tổng quát lên m số đó :)

Bài viết đã được chỉnh sửa nội dung bởi h.vuong_pdl: 27-03-2011 - 14:36

rongden_167


#5
Giang1994

Giang1994

    C'est la vie

  • Thành viên
  • 249 Bài viết
viện binh :( :( :( :)
1.với n tự nhiên, a,b dương. CmR:
$\sum\limits_{i=1}^{n}a_{1}^{2n+1} \geq (a_1a_2...a_n)^2(a_1+a_2+...+a_n)$
2. n,l tự nhiên không đồng thời bằng 0, $a_i$ dương
chứng minh $\sum\limits_{i=1}^{n} a_{i}^{n+l} \geq a_1a_2..a_n(a_{1}^{l}+a_{2}^{l} +...+ $ $:(^{l})$

Bài viết đã được chỉnh sửa nội dung bởi Giang1994: 27-03-2011 - 15:02

Don't let people know what you think


#6
anhtuanDQH

anhtuanDQH

    Thượng sĩ

  • Thành viên
  • 236 Bài viết

viện binh :Rightarrow :Rightarrow :Rightarrow :Rightarrow
1.với n tự nhiên, a,b dương. CmR:
$\sum\limits_{i=1}^{n}a_{1}^{2n+1} \geq (a_1a_2...a_n)^2(a_1+a_2+...+a_n)$

Tạm thời mình chỉ chén được bài 1 thôi .
Áp dụng BDT Chebyshev ta có :
$\sum\limits_{i=1}^{n}a_{1}^{2n+1} \geq \dfrac{1}{n} \sum\limits_{i=1}^{n} a_{1}\sum\limits_{i=1}^{n} a_{1}^{2n} $
Đến đây áp dụng AM-GM , ta có :
$\sum\limits_{i=1}^{n} a_{1}^{2n} \geq \ n( a_1 a_2...a_n)^2 $
$\Rightarrow DPCM $

Bài viết đã được chỉnh sửa nội dung bởi anhtuanDQH: 29-03-2011 - 21:27

Xăng có thể cạn, lốp có thể mòn..xong số máy số khung thì không bao giờ thay đổi

NGUYỄN ANH TUẤN - CHỦ TỊCH HIỆP HỘI
Hình đã gửi


#7
dark templar

dark templar

    Kael-Invoker

  • Hiệp sỹ
  • 3788 Bài viết

Tạm thời mình chỉ chén được bài 1 thôi .
Áp dụng BDT Chebyshev ta có :
$\sum\limits_{i=1}^{n}a_{1}^{2n+1} \geq \dfrac{1}{n} \sum\limits_{i=1}^{n} a_{1}\sum\limits_{i=1}^{n} a_{1}^{2n} $
Đến đây áp dụng AM-GM , ta có :
$\sum\limits_{i=1}^{n} a_{1}^{2n} \geq \ n( a_1 a_2...a_n)^2 $
$\Rightarrow DPCM $

Giải đc bài này rồi mà không giải đc bài dưới àh :Rightarrow=))
Sử dụng BĐT Chebyvsev+AM-GM,ta có:
$ \sum\limits_{i=1}^{n}a_{i}^{n+l} \ge \dfrac{1}{n}\left( \sum\limits_{i=1}^{n}a_{i}^n \right) \left( \sum\limits_{i=1}^{n}a_{i}^l \right) \ge \prod_{i=1}^{n}a_{i} \sum\limits_{i=1}^{n}a_{i}^{l}(Q.E.D)$
"Do you still... believe in me ?" Sarah Kerrigan asked Jim Raynor - Starcraft II:Heart Of The Swarm.

#8
Giang1994

Giang1994

    C'est la vie

  • Thành viên
  • 249 Bài viết

3.$ \sum\limits_{i=1}^{n}x_{i}^{n+1}\geq x_1x_2...x_n(x_1+x_2+...+x_n)$
--------------
đừng chê dễ quá ha!
--------------
:Rightarrow^_^:Rightarrow^_^:Rightarrow

các bạn làm kinh quá!
thực ra mấy bài này AM-GM là đc mà!
mình chém bài 3 thế này: mấy bài khác tương tự
có $x_1^{n+1}+x_1^{n+1}+x_2^{n+1}+...+x_n^{n+1} \geq (n+1) x_1.x_1x_2..x_n $
tương tự với các số khác rồi cộng lại là ra :Rightarrow

Don't let people know what you think


#9
anhtuanDQH

anhtuanDQH

    Thượng sĩ

  • Thành viên
  • 236 Bài viết

Giải đc bài này rồi mà không giải đc bài dưới àh :Rightarrow:Rightarrow
Sử dụng BĐT Chebyvsev+AM-GM,ta có:
$ \sum\limits_{i=1}^{n}a_{i}^{n+l} \ge \dfrac{1}{n}\left( \sum\limits_{i=1}^{n}a_{i}^n \right) \left( \sum\limits_{i=1}^{n}a_{i}^l \right) \ge \prod_{i=1}^{n}a_{i} \sum\limits_{i=1}^{n}a_{i}^{l}(Q.E.D)$

Ặc lúc đấy gần nửa đêm rùi còn chém gì nữa :Rightarrow

Xăng có thể cạn, lốp có thể mòn..xong số máy số khung thì không bao giờ thay đổi

NGUYỄN ANH TUẤN - CHỦ TỊCH HIỆP HỘI
Hình đã gửi





2 người đang xem chủ đề

0 thành viên, 2 khách, 0 thành viên ẩn danh